Difference between revisions of "2008 AMC 10B Problems/Problem 11"

(Solution)
(Solution)
(6 intermediate revisions by 6 users not shown)
Line 1: Line 1:
 
==Problem==
 
==Problem==
Postman Pete has a pedometer to count his steps. The pedometer records up to 99999 steps, then flips over to 00000 on the next step. Pete plans to determine his mileae for a year. On January 1 Pete sets the pedometer to 00000. During the year, the pedometer flips from 99999 to 00000 forty-four
+
Suppose that <math>(u_n)</math> is a [[sequence]] of real numbers satifying <math>u_{n+2}=2u_{n+1}+u_n</math>,
times. On December 31 the pedometer reads 50000. Pete takes 1800 steps per mile. Which of the following is closest to the number of miles Pete walked during the year?
 
 
 
(A) 2500 (B) 3000 (C) 3500 (D) 4000 (E) 4500
 
 
 
==Problem==
 
{{Suppose that <math>(u_n)</math> is a serquence of real numbers satifying <math>u_{n+2}=2u_{n+1}+u_n</math>,
 
  
 
and that <math>u_3=9</math> and <math>u_6=128</math>. What is <math>u_5</math>?
 
and that <math>u_3=9</math> and <math>u_6=128</math>. What is <math>u_5</math>?
  
(A) 40 (B) 53 (C) 68 (D) 88 (E) 104}}
+
<math>\mathrm{(A)}\ 40\qquad\mathrm{(B)}\ 53\qquad\mathrm{(C)}\ 68\qquad\mathrm{(D)}\ 88\qquad\mathrm{(E)}\ 104</math>
  
 
==Solution==
 
==Solution==
{{We konw that <math>u_6=128</math>, so we plug in <math>n=4</math> to get <math>128=2u_5+u_4</math>. We plug in <math>n=3</math> to get
+
Plugging in <math>n=4</math>, we get  
 
 
<math>u_5=2u_4+9</math>. Substituting gives
 
  
<math>128=5u_4+18 \rightarrow u_4=22</math>
+
<center><math>128=2u_5+u_4.</math></center>  
  
This gives <math>u_5=\frac{128-22}{2}=53</math>.
+
Plugging in <math>n=3</math>, we get
  
Answer B is the correct answer
+
<center><math>u_5=2u_4+9.</math></center>
  
NOTE: This is my (BOGTRO) solution, not the official one,
+
This is simply a system of two equations with two unknowns. Substituting gives <math>128=5u_4+18 \Longrightarrow u_4=22</math>, and <math>u_5=\frac{128-22}{2}=53 \longleftarrow \textbf{(B)}</math>.
 
 
and should be ignored in view of a better solution.}}
 
  
 
==See also==
 
==See also==
{{AMC10 box|year=2008|ab=B|num-b=9|num-a=11}}
+
{{AMC10 box|year=2008|ab=B|num-b=10|num-a=12}}
  
==See also==
+
[[Category:Introductory Algebra Problems]]
{{AMC10 box|year=2008|ab=B|num-b=10|num-a=12}}
+
{{MAA Notice}}

Revision as of 21:45, 13 November 2017

Problem

Suppose that $(u_n)$ is a sequence of real numbers satifying $u_{n+2}=2u_{n+1}+u_n$,

and that $u_3=9$ and $u_6=128$. What is $u_5$?

$\mathrm{(A)}\ 40\qquad\mathrm{(B)}\ 53\qquad\mathrm{(C)}\ 68\qquad\mathrm{(D)}\ 88\qquad\mathrm{(E)}\ 104$

Solution

Plugging in $n=4$, we get

$128=2u_5+u_4.$

Plugging in $n=3$, we get

$u_5=2u_4+9.$

This is simply a system of two equations with two unknowns. Substituting gives $128=5u_4+18 \Longrightarrow u_4=22$, and $u_5=\frac{128-22}{2}=53 \longleftarrow \textbf{(B)}$.

See also

2008 AMC 10B (ProblemsAnswer KeyResources)
Preceded by
Problem 10
Followed by
Problem 12
1 2 3 4 5 6 7 8 9 10 11 12 13 14 15 16 17 18 19 20 21 22 23 24 25
All AMC 10 Problems and Solutions

The problems on this page are copyrighted by the Mathematical Association of America's American Mathematics Competitions. AMC logo.png